The passage suggests that the author would be likely to agree with each of the following statements EXCEPT:

shafieiava on February 8, 2020

Answer choice C

Can someone explain why C is the correct answer choice? I feel like the last paragraph provides adequate support for this answer choice. Can someone please explain in detail what disqualifies this answer choice as something that author would not agree with? Is it the use of the term rate? Thanks in advance.

Reply
Create a free account to read and take part in forum discussions.

Already have an account? log in

shunhe on February 8, 2020

Hi @shafieiava,

Thanks for the question! The wording in this answer choice is very tricky and it’s what ultimately makes (C) the correct answer. So recall that under the utility maximization principle, people will try to maximize their expected utility, and so will do whatever action it is that maximizes that expected utility. Now let’s say there’s two classes of activities: lawful and unlawful. (C) tells us that the expected utility of lawful activities decreases. That means that some people who had a higher expected utility from doing lawful activities will now have a higher expected utility from doing unlawful activities. This, in turn, will lead to an increase in the crime rate. However, this is the opposite of what (C) says, which is that rate of deliberate crimes would decrease in the event of a decrease in the expected utility of lawful activities. Thus, the author would disagree with (C), and so (C) is the right answer. Hope this helps! Feel free to ask any other questions that you might have.